Q21

 
joyce.hau
Thanks Received: 0
Forum Guests
 
Posts: 12
Joined: May 20th, 2010
 
 
 

Re: PT57, S1, G4 - A charitable foundation awards grants

by joyce.hau Wed Dec 01, 2010 5:35 am

I don't understand how question 21 works. We are trying to fit 6 grants into 4 slots, and each slot has at least one grant in it, so aren't the only possible distributions as follows:
1-1-2-2
or 1-1-1-3

So how is it possible that you can have exactly 2 grants in JUST ONE of the four quarters?
 
finnsells
Thanks Received: 1
Forum Guests
 
Posts: 1
Joined: November 19th, 2010
 
This post thanked 1 time.
 
 

Re: PT57, S1, G4 - A charitable foundation awards grants

by finnsells Fri Dec 03, 2010 11:13 am

Actually the rules stipulate "No more than six grants are awarded" so you can have LESS than six. B is the only choice that can work w/out breaking the other rules while keeping just 1 quarter w two grants.
 
angela-a
Thanks Received: 0
Forum Guests
 
Posts: 2
Joined: August 22nd, 2011
 
 
 

Re: Q21

by angela-a Thu Sep 29, 2011 1:08 am

Can someone please explain to me why B is the answer choice and not D. I was able to confirm the could be true for both choices. Am obviously missing some thing. Could someone clarify. Thank.
 
yoohoo081
Thanks Received: 9
Elle Woods
Elle Woods
 
Posts: 66
Joined: March 16th, 2011
 
 
 

Re: Q21

by yoohoo081 Thu Sep 29, 2011 3:07 pm

angela-a Wrote:Can someone please explain to me why B is the answer choice and not D. I was able to confirm the could be true for both choices. Am obviously missing some thing. Could someone clarify. Thank.


* W is blue to show that it's a locked in condition*

B is answer because:
remember M? There HAS to be two according to the condition.

example why this is possible (since it's a COULD be true)
M
T W Y M
1 2 3 4

If neither Y nor M could go into T, that leaves T right?
And because of the 3rd condition (no grants same area are award same quarter or in consecutive) M HAS to go in 2 & 4.
If you see the diagram, it works if exactly 2 grants are awarded.

D is NOT the answer because:
M constraint again!

example why this is NOT possible

T
Y W M M
1 2 3 4

Uh oh... if Y & T are positioned according to what D tells you, M ends up together.

Just remember, TWO Ms
grants CANNOT BE on directly top/ bottom/ right/ left of each other
W= LOCKED IN 2

Hope this helped!
 
alexcyt1995
Thanks Received: 0
Vinny Gambini
Vinny Gambini
 
Posts: 5
Joined: April 24th, 2016
 
 
 

Re: Q21

by alexcyt1995 Thu May 12, 2016 5:26 am

Does anyone think the wording in this question stem is a little bit of confusing?

"Exactly 2 grants are awarded in just one of the 4 quarters". The correct way to understand this would be "only 1 quarter could contain 2 grants, the others can only contain 1". But could it also mean that "exactly 2 grants could be awarded once(one of the quarters), while the other 2 grants could only be awarded twice"?

Could anyone clarify this? Thanks!!!
User avatar
 
ManhattanPrepLSAT1
Thanks Received: 1909
Atticus Finch
Atticus Finch
 
Posts: 2851
Joined: October 07th, 2009
 
This post thanked 1 time.
 
 

Re: Q21

by ManhattanPrepLSAT1 Fri May 13, 2016 3:26 pm

Keep in mind that in your latter example there would be six grants awarded, which either means no quarter has exactly two grants awarded (3-1-1-1) or more than one quarter is going to have exactly two grants (2-2-1-1). Neither of these satisfies the new condition of this question.

Here's our setup for this one:

Image

The key on this question is to recognize that only one distribution is possible (2-1-1-1). That ensures that only one of the four areas repeats, and we know which one ... medical services.

(A) cannot be true. We know that youth services will only be awarded a grant in one quarter.
(C) cannot be true. We know that wildlife preservation will only be awarded a grant in the second quarter.
(D) cannot be true. If youth services and theater arts are awarded grants in the first quarter, then both medical services grants would be awarded in the third and fourth quarters and would be consecutive.
(E) cannot be true for the same reason as (D). This would require that both medical services grants would be awarded in consecutive quarters (third and fourth).

That leaves answer choice (B), which could be true:

Image
 
SelenaY172
Thanks Received: 0
Vinny Gambini
Vinny Gambini
 
Posts: 1
Joined: August 03rd, 2017
 
 
 

Re: Q21

by SelenaY172 Thu Aug 03, 2017 11:49 pm

alexcyt1995 Wrote:Does anyone think the wording in this question stem is a little bit of confusing?

"Exactly 2 grants are awarded in just one of the 4 quarters". The correct way to understand this would be "only 1 quarter could contain 2 grants, the others can only contain 1". But could it also mean that "exactly 2 grants could be awarded once(one of the quarters), while the other 2 grants could only be awarded twice"?

Could anyone clarify this? Thanks!!!


Hi alexcyt1995 I had the same problem with you when I was doing my timed PT. But now I think I get it.

"Grants" and "areas" are different. Medical services, theater arts etc., these are areas, not grants. The question says exactly two grants are awarded in just on quarter, not exactly two areas are awarded in just one quarter. So your second interpretation cannot be applied here, since we can hardly say which grant (instead of area) can be literally awarded twice. If you change "grants" to "apples", and "areas" to "colors", then the wording should be clearer. You cannot say that any single apple can be given twice in different quarters.

Hope that helps.